Đến nội dung

hoangson2598 nội dung

Có 321 mục bởi hoangson2598 (Tìm giới hạn từ 29-04-2020)



Sắp theo                Sắp xếp  

#608880 Thể khảm là gì?

Đã gửi bởi hoangson2598 on 14-01-2016 - 00:01 trong Các môn tự nhiên (Vật lý, Hóa học, Sinh học, Công nghệ)

Ai giải thích hộ mình với! Ai hiểu thì giúp chứ đừng tra google!




#606320 Giải Hệ Phương Trình

Đã gửi bởi hoangson2598 on 31-12-2015 - 16:59 trong Phương trình - hệ phương trình - bất phương trình

áp dụng bunhia sai rồi  :wacko:

Áp dụng $(a^2+b^2)(c^2+d^2)\geq (ac+bd)^2$

suy ra: 

$((\sqrt{mn})^2+1)((\sqrt{\frac{m}{n}})^2+1)\geq (\sqrt{mn}.\sqrt{\frac{m}{n}}+1)^2=(m+1)^2$




#606127 Giải Hệ Phương Trình

Đã gửi bởi hoangson2598 on 30-12-2015 - 15:53 trong Phương trình - hệ phương trình - bất phương trình

bất đẳng thức sai rồi bạn 

Áp dụng bunhia: 

$(mn+1)(\frac{m}{n}+1)\geq (m+1)^2\Leftrightarrow \frac{1}{(m+1)^2}\geq \frac{1}{mn+1}(\frac{n}{m+n})$

Tương tự ta có $\frac{1}{(n+1)^2}\geq \frac{1}{mn+1}(\frac{m}{m+n})$

Cộng vào suy ra 

$\frac{1}{(m+1)^2}+\frac{1}{(n+1)^2}\geq \frac{1}{mn+1}$




#606064 Giải Hệ Phương Trình

Đã gửi bởi hoangson2598 on 30-12-2015 - 00:12 trong Phương trình - hệ phương trình - bất phương trình

giải hpt: 

 

$\frac{2}{(\sqrt{x}+\sqrt{y})^{2}}+\frac{1}{x+\sqrt{y(2x-y)}}=\frac{2}{y+\sqrt{X(2x-y)}}$ 

$2(y-4)\sqrt{2x-y-3}-(x-6)\sqrt{x+y+1}=3(y-2)$

chia cả hai mẫu của phương trình 1 cho y rồi đặt $\frac{x}{y}=a$ ta được

$\frac{1}{(\sqrt{a}+1)^2}+\frac{1}{(\sqrt{2a-1}+1)^2}=\frac{1}{1+\sqrt{a(2a-1)}}$

Đặt $\sqrt{a}=m$, $\sqrt{2a-1}=n$ ta được:

$\frac{1}{(m+1)^2}+\frac{1}{(n+1)^2}=\frac{1}{1+mn}$ 

Một bđt quen thuộc suy ra 

$m=n\Leftrightarrow a=2a-1\Leftrightarrow a=1\Leftrightarrow x=y$




#605558 Bình chọn Thành viên nổi bật 2015 của Diễn đàn toán học

Đã gửi bởi hoangson2598 on 27-12-2015 - 19:22 trong Thông báo tổng quan

em thấy mấy bạn sôi nổi đăng với cả comment trong mấy cái trạng thái ấy cũng ấn tượng mà!




#604089 $a^{3}b^{2}+b^{3}c^{2}+c^{3...

Đã gửi bởi hoangson2598 on 19-12-2015 - 23:25 trong Bất đẳng thức và cực trị

Cho các số dương $a$,$b$,$c$>0. Chứng minh:

$a^{3}b^{2}+b^{3}c^{2}+c^{3}a^{2}>a^{2}b^{3}+b^{2}c^{3}+c^{2}a^{3}$

 

Giả sử $a\geq b\geq c:$>0

$\Rightarrow$BĐT$\Leftrightarrow$ Sa(a-b)+Sb(b-c)+Sc(c-a)$\geq 0$ với Si>0

tách b-c=-((a-b)+(c-a)) ghép dần sẽ có đpcm.

Dấu = xảy ra $\Leftrightarrow$a=b=c>0

Thay a=1, b=2, c=3  thì VT=103 <125=VP Vô lý!!




#603893 $P=x^2+y^2+z^2$

Đã gửi bởi hoangson2598 on 18-12-2015 - 23:55 trong Bất đẳng thức và cực trị

 

 

Từ giả thiết$=>\frac{(x+y+z)^3-1}{3(x+y+z)}=xy+yz+zx$

$=>P=(x+y+z)^2-\frac{2(x+y+z)^3-2}{3(x+y+z)}=t^2+\frac{2t^3-2}{3t}$

Ta có: $P'(t)=\frac{2(t^3-1)}{2t^2}=0<=>t=1$

$=>P\geqslant 1$

 

Từ giả thiết$=>\frac{(x+y+z)^3-1}{3(x+y+z)}=xy+yz+zx$

$=>P=(x+y+z)^2-\frac{2(x+y+z)^3-2}{3(x+y+z)}=t^2+\frac{2t^3-2}{3t}$

Ta có: $P'(t)=\frac{2(t^3-1)}{2t^2}=0<=>t=1$

$=>P\geqslant 1$

Nhầm một chút nhỉ!




#603529 Chứng minh BĐT bằng phương pháp biến đổi tương đương

Đã gửi bởi hoangson2598 on 16-12-2015 - 22:31 trong Bất đẳng thức và cực trị

Cho các số thực x, y, z thỏa mãn điều kiện x^2+y^2+z^2=9.

Chứng minh rằng  2(x+y+z)-xyz<=10

Đề thi VMO, bài này dùng dồn biến!!

Còn biến đổi tương đương thì đợi mình học thêm một thời gian nữa!!




#603017 $\frac{2z^2+zx}{(x+\sqrt{yz}+y)^2...

Đã gửi bởi hoangson2598 on 13-12-2015 - 19:11 trong Bất đẳng thức và cực trị

Cho x, y, z là các số thực dương. Chứng minh rằng:

$\frac{2x^2+xy}{(y+\sqrt{zx}+z)^2}+\frac{2y^2+yz}{(z+\sqrt{xy}+x)^2}+\frac{2z^2+zx}{(x+\sqrt{yz}+y)^2}\geq 1$




#602440 $\begin{cases} & x^3+3xy^2=-49 \\ &...

Đã gửi bởi hoangson2598 on 09-12-2015 - 23:04 trong Phương trình - hệ phương trình - bất phương trình

Giải hệ pt:

 

$\begin{cases} &  x^3+3xy^2=-49 \\  &  x^2-8xy+y^2=8y-17x \end{cases}$

Ở đây 

http://diendantoanho...endmatrixright/




#602427 Chứng minh rằng $C_{m}^{0}C_{n}^{k...

Đã gửi bởi hoangson2598 on 09-12-2015 - 22:33 trong Tổ hợp - Xác suất và thống kê - Số phức

giả sử k,m,n là 3 số tự nhiên thỏa mãn $m\leq k\leq n$. Chứng minh rằng:

 $C_{m}^{0}C_{n}^{k}+C_{m}^{1}C_{n}^{k-1}+...+C_{m}^{m}C_{n}^{k-m}=C_{n+n}^{k}$

Ta luôn có:

$(1+x)^m.(1+x)^n=(1+x)^{m+n}$

Xét hệ số của $x^k$ trong khai triển $(1+x)^{m+n}$ chính là vế phải của biểu thức

Xét hệ số của $x^k$ trong khai triển $(1+x)^m.(1+x)^n$ chính là vế trái của biểu thức

Suy ra điều cần chứng minh!




#602302 Bất đẳng thức chuẩn bị cho kì thi THPTQG 2015-2016

Đã gửi bởi hoangson2598 on 09-12-2015 - 00:23 trong Bất đẳng thức và cực trị

Cho x, y, z>0, tìm GTLN

$P=\frac{1}{2\sqrt{x^{2}+y^{2}+z^{2}+1}}-\frac{1}{(x-1)(y-1)(z-1)}$

Hình như là (x+1)(y+1)(z+1) chứ nhỉ?

http://diendantoanho...21-frac2a1b1c1/




#601818 KÉN DÂU CHO VMF

Đã gửi bởi hoangson2598 on 05-12-2015 - 22:21 trong Góc giao lưu

Kính chào các anh em VMF

 

Như chúng ta đã biết, admin VMF E Galois là 1 trong những tài năng xuất sắc nhất của toán học Việt Nam vào thời điểm hiện tại. Tuy nhiên, sau bao nhiêu năm miệt mài cống hiến cho VMF, hiện chỉ còn vài ngày nữa là admin này sẽ bước qua tuổi 3 xị. và khổ là " Đã bao mùa khoai sọ chưa này nọ cùng ai", hẳn mỗi con dân của VMF đều mong E Galois sẽ sớm có người nâng khăn sửa túi để chuyên tâm cống hiến nhiều hơn cho VMF.
 

 

Nhằm tri ân những cống hiến quên mình của E Galois cho VMF, supermember quyết định mở vận động hành lang, ai có chị gái thì giới thiệu chị gái, ai có em gái thì giới thiệu em gái, ai có bà cô bà dì bà vú gì thì cứ giới thiệu, miễn sao trong khoảng tin cậy [ 20 ; 30] , giới tính nữ rõ ràng, ngay cả xinh cỡ Hương Giang Idol cũng loại từ vòng gửi xe. Không cần phải chân dài tới nách, chỉ cần lông nách dài tới chân.

 

Hình thức: post ảnh người giới thiệu lên kèm theo thông tin : tuổi , tên, FB, Zalo, Yahoo, địa chỉ, nghề nghiệp.....số đo 3 vòng. Ưu tiên các ứng viên có hộ khẩu ở miền Bắc. Vì yêu xa cũng khổ lắm =]], mà đợi chờ thì không bao giờ là hạnh phúc =]]

 

Vì 1 VMF không FA, hãy giúp 1 tay các bạn nhé =]]

Sau đây em xin giới thiệu đến ad Thế một người khá là tuyệt vời!

Cao 1m65, da hơi giống gỗ lim, dễ dàng nguỵ trang khi trời chập tối, tóc đen xì, không 1 cộng bạc, sức đề kháng vô cùng khủng khiếp, tương truyền là chưa có ai nghe thấy 1 tiếng ho của thầy bao giờ! Râu ria không có (hoặc là do lẫn với màu da), thị giác và thính giác vô cùng tốt, không cần nhìn mà vẫn có thể biết học sinh đang nghịch cái gì trong giờ học. Về chiến thuật thì có thể coi thầy giống như rô sề mô di nhô với những mưu kế vô cùng thâm độc khiến học sinh tức muốn đâm đầu vào tường mà không được (đâm vào hỏng tường là lại đi lao động)............. 

Và một chỉ tiêu vô cùng quan trọng là lông lách dài đến chân thì thầy đã vượt chỉ tiêu!

Em giới thiệu thầy chủ nhiệm của em không phải là để lấy làm vợ mà là để cho thầy Thế liên hệ nếu cần tư vấn. Thầy em năm nay 40 và mới lấy vợ năm ngoái!!!!




#601161 $\max P=\frac{2(b+c-1)}{a}+\frac...

Đã gửi bởi hoangson2598 on 01-12-2015 - 23:33 trong Bất đẳng thức và cực trị

Cho $a, b, c$ thực không âm thỏa mãn : $b^2+c^2+1=9a^2$
Tìm $\max P=\frac{2(b+c-1)}{a}+\frac{bc}{a^3}$

Chia cả hai vế của đk cho $a^2$

Đặt $\frac{b}{a}=x$, $\frac{c}{a}=y$, $\frac{-1}{a}=z$

suy ra P=$2(\frac{b}{a}+\frac{c}{a}-\frac{1}{a})+\frac{b}{a}.\frac{c}{a}.\frac{1}{a}=2(x+y+z)-xyz$

Đưa về bài tìm max  P=$2(x+y+z)-xyz$ với $x^2+y^2+z^2=9$

Rồi sau đó tiếp tục giải như bài sau, nó làm dồn biến lằng nhằng mình cũng chả biết, cứ copy vào đây cho bạn!!

12336094_1531284743863619_1331904264_n.j




#599618 $\frac{xy}{z}+\frac{xz}{y...

Đã gửi bởi hoangson2598 on 22-11-2015 - 20:51 trong Số học

Tìm nghiệm nguyên của PT: $\frac{xy}{z}+\frac{xz}{y}+\frac{yz}{x}=3$

Có cho x, y, z >0 để áp dụng bđt không?




#598206 Chứng minh: $A,M,N$ thẳng hàng

Đã gửi bởi hoangson2598 on 13-11-2015 - 22:11 trong Hình học

Cho tam giác $ABC$. Gọi $I,J,M$ lần lượt là trung điểm của $AB,AC$ và $IJ$. Đường tròn ngoại tiếp tam giác $AIJ$ có tâm $O$. $D$ là điểm đối xứng với $A$ qua $O$. Gọi $N$ là chân đường vuông góc hạ từ $D$ xuống $BC$. CMR: $A,M,N$ thẳng hàng. 

AD là đường kính

Suy ra DI vuông góc AB và DJ vuông góc AC

Suy ra D là tâm đường tròn ngoại tiếp ABC

Suy ra N là trung điểm BC

Gọi N' là giao điểm của AM với BC thì chứng minh được N' là trung điểm BC

Suy N' trùng với N

Suy ra AMN thẳng hàng




#595643 $\sqrt[3]{x^2-1}-\sqrt{x^3+2}+x=0$

Đã gửi bởi hoangson2598 on 27-10-2015 - 19:43 trong Phương trình - hệ phương trình - bất phương trình

Từ pt1 =>$(\sqrt{x^2+2}+y)(\sqrt{x^2+2}-y^2+1)=0$

 

Làm ntn mà ra đc pt kia vậy???????? Bạn viết rõ ra được ko?

Viết nhầm nhá!

Chỗ kia là $y^2$ chứ không phải là $y^3$




#595549 $\sqrt[3]{x^2-1}-\sqrt{x^3+2}+x=0$

Đã gửi bởi hoangson2598 on 26-10-2015 - 23:17 trong Phương trình - hệ phương trình - bất phương trình

Với dạng nghiệm khủng thế này em nghĩ đề bài $\sqrt[3]{x^2-1}-\sqrt{x^3+2}+x=0$ có vấn đề.

Bài gốc nó là bài hệ

$x^2-(y^2-y-1)\sqrt{x^2+2}-y^3+y+2=0$

$\sqrt[3]{y^2-3}-\sqrt{xy^2-2x+2}+x=0$




#595509 $\sqrt[3]{x^2-1}-\sqrt{x^3+2}+x=0$

Đã gửi bởi hoangson2598 on 26-10-2015 - 21:16 trong Phương trình - hệ phương trình - bất phương trình

Giải phương trình

$\sqrt[3]{x^2-1}-\sqrt{x^3+2}+x=0$




#592041 $\sum \frac{1}{(a+2b)^{2}}...

Đã gửi bởi hoangson2598 on 04-10-2015 - 14:43 trong Bất đẳng thức và cực trị

AD BĐT Cauchy - Schwarz:

 

$\sum \frac{1}{(a+2b)^2} \geq \frac{9}{5(a^2+b^2+c^2)+4(ab+bc+ca)} \geq \frac{9}{9(ab+bc+ca} \geq \frac{1}{ab+bc+ca}$

??

 $\frac{9}{5(a^2+b^2+c^2)+4(ab+bc+ca)} \geq \frac{9}{9(ab+bc+ca}$




#592039 $P=\sum \sqrt[3]{4(x^{3}+y^{3})}...

Đã gửi bởi hoangson2598 on 04-10-2015 - 14:40 trong Bất đẳng thức - Cực trị

Cho x,y,x>0. Tìm min: $P=\sum \sqrt[3]{4(x^{3}+y^{3})} + 2\sum \frac{x}{y^{2}}$

Dễ dàng chứng minh được $4(x^3+y^3)\geq (x+y)^3$ bằng biến đổi tương đương

Suy ra  $\sum \sqrt[3]{4(x^{3}+y^{3})} \geq x+y$

Tương tự rồi cộng vào ta được $P\geq2(x+y+z+\frac{x}{y^2}+\frac{y}{z^2}+\frac{z}{x^2})\geq12$

Dấu = xảy ra khi $x=y=z=1$




#591982 $P=xy + yz + zx$

Đã gửi bởi hoangson2598 on 04-10-2015 - 09:32 trong Bất đẳng thức và cực trị

Cho $x,y,z$ là nghiệm của hệ 2 phương trình:

$x^2  + xy + y^2  = 3 $ và $y^2  + yz + z^2  = 16 $

 
Tìm max của: $P=xy + yz + zx$



#591952 $2x^2+3xy=3y-13$

Đã gửi bởi hoangson2598 on 04-10-2015 - 00:08 trong Phương trình - hệ phương trình - bất phương trình

Giải hệ phương trình:

$2x^2+3xy=3y-13$

$3y^2+2xy=2x+11$




#591947 $x^3-3x^2-6x+2(x+2)\sqrt{x+2}<0$

Đã gửi bởi hoangson2598 on 03-10-2015 - 23:29 trong Phương trình - hệ phương trình - bất phương trình

rút cái -3x^2-6x ra rồi nhóm với x+2 coi sao

Không được!




#591943 $x^3-3x^2-6x+2(x+2)\sqrt{x+2}<0$

Đã gửi bởi hoangson2598 on 03-10-2015 - 23:14 trong Phương trình - hệ phương trình - bất phương trình

Giải bất phương trình sau:

$x^3-3x^2-6x+2(x+2)\sqrt{x+2}<0$